LSAT and Law School Admissions Forum

Get expert LSAT preparation and law school admissions advice from PowerScore Test Preparation.

User avatar
 erikariasss
  • Posts: 4
  • Joined: Feb 09, 2021
|
#88085
Hi, can I please get an explanation as to why (D) is a wrong answer choice.
User avatar
 Stephanie Oswalt
PowerScore Staff
  • PowerScore Staff
  • Posts: 826
  • Joined: Jan 11, 2016
|
#88123
erikariasss wrote: Fri Jun 18, 2021 5:18 pm Hi, can I please get an explanation as to why (D) is a wrong answer choice.
Hi erikariasss,

Thanks for the post! Please refer to the official explanation on page 1 of this thread: viewtopic.php?f=707&t=9242. The explanation mentions why D is an irrelevant answer choice, as do a few of the posts in the discussion on that same page. :)

Let us know if this helps, or if you still have further questions. Thanks!
User avatar
 Rosepose24
  • Posts: 13
  • Joined: Feb 07, 2021
|
#88129
Hi I read through the initial explanation. I got this question correct but I don't agree with the reasonings written to get there.

This is my thought process:

ID Conclusion- (businesses who will have such accidents) - will now install adequate enviro guards.

I felt the conclusion did follow from the premises. Seeking a prephase that left the premises alone - profits still valued! fines still out cost the safety measures!... yet the conclusion challenged.

prephase: something that lets all the premises be true, but maybe they do not install "adequate" guards.

Answer choice A: if this were true, all the premises could stand true and the business would (as it follows in the stimulus) install safety measures matching those of the risk level they estimate. So, if they underestimate, then their safety measures maybe reflect that. This challenges the conclusion because they don't necessarily install ADEQUATE guards for an underestimated risk.

Is this correct? I felt that the explanation on this thread left that key word out altogether.
 Robert Carroll
PowerScore Staff
  • PowerScore Staff
  • Posts: 1787
  • Joined: Dec 06, 2013
|
#88310
Rose,

The argument in the stimulus is not valid because it depends on businesses correctly judging that the cost of fines will exceed the cost of measures to prevent the accidents that would result in the fines. If a business has perfect knowledge, and if the author is right that the fines will exceed the cost of safety measures, then businesses that value profits will always implement the safety measures, because that's the cheaper option. If businesses do not have perfect knowledge, though, they may not realize that their failure to spend a certain amount of money on safety measures will make them pay more in fines than the safety measures would have cost. Answer choice (A) exploits that problem.

I think you got that, and if the initial post didn't make that clear enough, I hope my post has!

Robert Carroll

Get the most out of your LSAT Prep Plus subscription.

Analyze and track your performance with our Testing and Analytics Package.